does anybody know the answer to this question?

Does Anybody Know The Answer To This Question?

Answers

Answer 1

Answer: 56 yd

Step-by-step explanation: if you continue the 8 yd line across, it would show that the missing side is 4yd less than the 13 yd side, meaning the missing side is 9 yd long. So after adding 7 + 13+ 15+ 4 + 8 + 9= 56 yd


Related Questions

find the volume
6ft
4ft
3ft
(i know it’s not 72)

Answers

Answer:

[tex] \fbox{Volume \: of \: pyramid \: = 24 ft³}[/tex]

Step-by-step explanation:

Given:

shape of object = Pyramidal

length of base l= 4ft

width of base w = 3ft

height of pyramid h = 6 ft

To find:

volume of pyramid = ?

Solution:

[tex]Volume \: of \: pyramid= \frac{l \times w \times h}{3} \\ Volume \: of \: pyramid= \frac{6 \times 4 \times 3}{3} \\ Volume \: of \: pyramid= \frac{24 \times \not3}{ \not3} \\ Volume \: of \: pyramid= 24 \: {ft}^{3} [/tex]

[tex] \fbox{Volume \: of \: pyramid \: = 24 ft³}[/tex]

Thanks for joining brainly community!

I need help please and thank you and show work

Answers

Answer:

Ivy is correct

Step-by-step explanation:

the perimeter of a parallelogram can be written as P= 2(a+b), where a and b are the lengths of the sides of the parallelogram.
since the parallelogram has been dilated by a scale factor of 0.6, we multiple the original side lengths by 0.6.
12 × 0.6 = 7.2
16 × 0.6 = 9.6
Plug into the equation
P=2(7.2+9.6)
P=2(16.8)

P= 33.6

You can also solve by plugging in the original values and multiplying the answer by 0.6.
P=2(12+16)(0.6)
P=2(28)(0.6)
P=(56)(0.6)

P=33.6

Which two sets of events are most likely independent?
O A. Event A: A student makes the drama team.
Event B. The student likes macaroni and cheese.
B. Event A: A student gets enough sleep.
Event B. The student is alert in class.
C. Event A: A car is blue.
Event B. The car lasts over 20 years.
D. Event A: A woman buys cat toys.
Event B: The woman has a cat.

Answers

Answer:

A - Event A: A student makes the drama team. / Event B. The student likes macaroni and cheese.C - Event A: A car is blue. / Event B. The car lasts over 20 years.

What is the midpoint of XY if X is located at (-12, 8) and Y is located at (10, 17)
A. (-2. 25)
B. (-1, 12 5)
C. (-2, 11.5)
D. (32, 26)
E. None of the above

Answers

Answer: B

Step-by-step explanation:

What is the area of this triangle?
1375 ft
1500ft
3300ft
4500ft

Answers

Answer:

probably 20 man think about it

Step-by-step explanation:

1/5c= 2 (whole number) 1/2


what is it

you have to divide them

ill also give the smartest
PLEASE

Answers

The value of c in the equation, 1/5c = 2 1/2 when divided is 12 1/2.

What are fractions?

Fractions are numbers that are non integers. They are ususally made up of numerators and denominators. The numerator is the number above and the denominator is the number below.

What is the value of c?

In order to determine the value of c, divide 2 1/2 by 1/5

2 1/2 ÷ 1/5

Convert the mixed fraction to an improper fraction

5/2 x 5 = 25/2 = 12 1/2

To learn more about the division of fractions, please check: https://brainly.com/question/25779356

What is the y-intercept of the line shown? A. 2 B. -2 C. 4 D. -4

Answers

The y-intercept will be where it crosses the y-axis. The y-axis is the line that goes up and down also know a vertically.

A wheel with the 31 in. diameter has 651 rotations. Reflect on how different wheel sizes would change your answer. What would happen to the number of rotations if the circumference of the wheels were increased by 20%? For the wheel with 31 in. diameter, multiply its circumference 97.4 in. by 1.2 =________.

Answers

The number of rotations is reduced to 542.501 when the diameter of wheel is increased by 20 %.

How to compare rotations between two wheels of different diameter

The quantity of rotations (n), no unit, done by a wheel is equal by total traveled distance (s), in inches, divided by its circumference (p), in inches:

[tex]n = \frac{s}{p}[/tex]  

[tex]n = \frac{s}{\pi\cdot D}[/tex]   (1)

Where D is the diameter of the wheel, in inches.

If we know that n = 651 and D = 31 in, then the travelled distance is:

s = (651) · π · (31 in)   (2)

s ≈ 63400.481 in

The travelled distance of the 31-in diameter wheel with 651 rotations is approximately 63400.481 inches.

By (2) we understand that the travelled distance is directly proportional to the diameter. Hence, if the diameter of the wheel is increased by 20 %, then we must multiply the diameter by 1.2 and divide the travelled distance by this result:

s ≈ 1.2 · π · (31 in)

s ≈ 116.867 in

[tex]n = \frac{63400.481\,in}{116.867\,in}[/tex]

[tex]n = 542.501[/tex]

The number of rotations is reduced to 542.501 when the diameter of wheel is increased by 20 %. [tex]\blacksquare[/tex]

To learn more on circumferences, we kindly invite to check this verified question: https://brainly.com/question/4268218

Pls help me !!
a sector of a circle has a central angle measure of 90°, and an area of 7 square inches. what is the area of the entire circle?

area of the circle = __ square inches

Answers

Answer:  28

Explanation:

90° is 90/360 = 1/4 of a full 360° rotation. Meaning that if the central angle of the pizza slice is 90°, then we have four equal slices.

One slice is 7 square inches, so all four slices combine to 7*4 = 28 square inches

True or False


1. A solution to 8 = -x + 10 is 2

Answers

Answer:

True

Step-by-step explanation:

So, Lets find out if the solution is true or not, So first let us find the value of x.

⇒ 8 = -x + 10

We can rewrite the equation as,

⇒ -x + 10 = 8

Subtracting 10 from both sides we get :

⇒ -x + 10 – 10 = 8 – 10

⇒ -x = -2

Multiplying both sides by (-1) :

⇒ -1(-x) = -1(-2)

x = 2

Therefore,

The statement "A solution to 8 = -x + 10 is 2" is true

Hey ! there

Answer:

True

Step-by-step explanation:

In this question we are given with an equation that is 8 = -x + 10 and we have also given its solution that is 2 .And We are asked to tell whether the solution of given equation is true or false .

For finding that we must have to solve the equation .

Solution : -

[tex]\longrightarrow \qquad \: 8 = -x + 10[/tex]

Step 1 : Flipping the equation so that there's ease in solving :

[tex] \longrightarrow \qquad \: - x + 10 = 8[/tex]

Step 2 : Subtracting with 10 on both sides :

[tex] \longrightarrow \qquad \: - x + \cancel{10 }- \cancel{ 10} =8 - 10[/tex]

On further calculations , We get :

[tex] \longrightarrow \qquad \: - x = - 2[/tex]

Step 3 : Multiplying with -1 on both sides :

[tex] \longrightarrow \qquad \: - x \times - 1 = - 2 \times - 1[/tex]

We get :

[tex] \longrightarrow \qquad \: \blue{\underline{\boxed{ \frak{x = 2}}}}[/tex]

Therefore , it's TRUE that 2 is the solution of given equation .

Alternative Solution : -

There's another easy way to check whether 2 is solution of given equation or not . This is done by substituting value of x that is 2 in given equation that is 8 = -x + 10 . So substituting value of x ( solution ) in given equation :

8 = - ( 2 ) + 10

8 = -2 + 10

8 = 8

L.H.S = R.H.S

As we can see that left side is equal to right side .

Therefore , it's TRUE that 2 is the solution of given equation .

#Keep Learning

find the equation of the line drawn in the coordinate system below​

Answers

Answer:

y=-2/1+4

or y=-2+4

Step-by-step explanation:

to find the slope, you count from one point of intersection to the next and the way it works is y/x so the line from one point to the next goes down 2 (making it negative also) and over 1, so -2/1 is your slope, and for the b part.. its just where the line intersects on the y axis (the line that goes up and down) so it would be 4

i do not know this answer

Answers

The value of the provided function 14g²,  at the point g equal to negative 1 is 14, when this value is put in the function.

How to evaluate the value of a function at a point?

To find the value of a function at a particular point, the value of the point has to be put on the place of a variable in the function and simplify the equation.

To evaluate the value of a function at a particular point, follow the steps given below.

Write down the function.Put the value of point in place of variable. Simplify the equation to find the value.

The function given in the problem is,

[tex]f(g)=14g^2[/tex]

Here, the value of the function has to evaluate at g=-1. Put the value of g in the above function as,

[tex]f(-1)=14(-1)^2\\[/tex]

The square of -1 is 1. Thus,

[tex]f(-1)=14\times1\\f(-1)=14[/tex]

Hence, the value of the provided function 14g²,  at the point g equal to negative 1 is 14, when this value is put in the function.

Learn more about the value of a function here;

https://brainly.com/question/875118

#SPJ1

solve this 6(x+4)+1=x+5

Answers

6x + 24+1= x+5
6x-x+25-5=0
5x+20=0
5x=-20
x= -20/5= -4
Answer:

[tex] \sf :\implies \quad {\pmb{x = - 4}}[/tex]

Solution:

Let's solve the given equation step by step ;

[tex] \sf :\implies \quad 6(x + 4) + 1 = x + 5[/tex]

The first step is to distribute the equation:

[tex] \sf :\implies \quad (6 \times x ) + (6 \times 4) + 1 = x + 5[/tex]

[tex] \sf :\implies \quad 6x + 24 + 1 = x + 5[/tex]

[tex] \sf :\implies \quad 6x + 25 = x + 5[/tex]

Second step is to combine the like terms:

[tex] \sf :\implies \quad 6x + 25 - x = 5[/tex]

[tex] \sf :\implies \quad( 6x - x) + 25 = 5[/tex]

[tex] \sf :\implies \quad 5x + 25 = 5[/tex]

Now, subtract 25 from both sides :

[tex] \sf :\implies \quad 5x + 25 - 25 = 5 - 25[/tex]

[tex] \sf :\implies \quad 5x = - 20[/tex]

Divide both sides by 5:

[tex] \sf :\implies \quad \dfrac{5x}{5} = - \dfrac{20}{5} [/tex]

[tex] \sf :\implies \quad \boxed{ \pmb{\sf x = - 4}}[/tex]

Hence, your answer is ; x = -4

Mr. Blue's first year salary is $30,000 and it doubles each year. What is the explicit
rule for the sequence in simplified form? what is the recursive rule?

Answers

The explicit rule for the sequence in simplified form is y = 30000(2)^n

How to write the exponential function

The standard exponential equation is expressed as:

y = ab^n

Given the following parameters

a is the initial values = 30,000
If the amount doubles each year then b = 2

Substitute the parameters

y = 30000(2)^n

Hence the explicit rule for the sequence in simplified form is y = 30000(2)^n

Learn more on explicit rule here: https://brainly.com/question/2059043

Answer: Explicit Rule: a_n=30,000 • 2^n-1 Recursive Rule: a_n = 2a_n-1; a_1 =30,000

Step-by-step explanation: the Explicit Rule for a geometric sequence is a_n = r • a_n -1 and the Recursive Rule is a_n = a_1 • a_n-1

a_1 is the first term and r is the common ration. In this case, the first term is 30,000 as stated in the question and the common ration is 2 since it doubles each time. So substitute those numbers into the formulas and that’s what you’ll get! Hope this helps! God bless you all!!

The first three steps in writing f(x) = 40x 5x2 in vertex form are shown. write the function in standard form. f(x) = 5x2 40x factor a out of the first two terms. f(x) = 5(x2 8x) form a perfect square trinomial. (eight-halves) squared = 16 f(x) = 5(x2 8x 16) – 5(16) what is the function written in vertex form?

Answers

The function f(x) = 5x² + 40x is a parabola. Then the vertex of the parabola will be at (–4, –80).

What is the parabola?

It is the locus of a point that moves so that it is always the same distance from a non-movable point and a given line. The non-movable point is called focus and the non-movable line is called the directrix.

The function f(x) = 5x² + 40x

Then add and subtract 80, then we have

f(x) = 5x² + 40x + 80 – 80

f(x) = 5(x² + 8x + 16) – 80

f(x) = 5(x + 4)² – 80

The vertex of the parabola will be at (–4, –80).

More about the parabola link is given below.

https://brainly.com/question/8495504

A $70,000 mortgage is $629.81 per month. What was the percent and for how many years?
9%, 20 years
9%, 30 years
7%, 20 years
9%, 25 years

Answers

The interest percent and the maturity period of the $70,000 mortgage with a monthly payment of $629.81 are A. 9%, 20 years.

How can the monthly payments be calculated?

The monthly payments can be computed using the present value annuity table or formula.

However, to compute the percent interest and the maturity period, we can use an online finance calculator as below by inputting the calculated monthly payments.

Thus, at an interest rate of 9% for 20 years and a monthly payment of $629.81, the present value of $70,000 will be equal.

Data and Calculations;

N (# of periods) = 240 (20 years x 12)

I/Y (Interest per year) = 9%

PMT (Periodic Payment) = $629.81

FV (Future Value) = $0

Results:

PV = $70,000.20

Sum of all periodic payments = $151,154.40 ($629.81 x 240)

Total Interest $81,154.20

The interest percent and the maturity period of the $70,000 mortgage with a monthly payment of $629.81 are A. 9%, 20 years.

Learn more about periodic payments at https://brainly.com/question/24244579

A machine is supposed to fill bags with an average of 19.2 ounces of candy. The manager of the candy factory wants to be sure that the machine does not consistently underfill or overfill the bags . So the manager plans to conduct a significance test at the alpha = 0.10 significance level of H_{0} / mu = 19.2; H a : mu ne19.2 where mu = the true mean amount of candy (in ounces) that the machine put in all bags filled that day. The manager takes a random sample of 75 bags of candy produced that day and weighs each bag. The sample mean weight for the bags of candy was 19.28 ounces and the sample standard deviation was 0.81 ounce.
Find the P-value.​

Answers

Answer: c. Step-by-step explanation:

c = y + a solve for y

Answers

Answer:

[tex]\boxed{\sf{y=c-a}}[/tex]

Step-by-step explanation:

To solve for y, isolate it on one side of the equation.

c=y+a

First, switch sides.

c=y+a

y+a=c

Then, you subtract by a from both sides.

y+a-a=c-a

Solve.

[tex]\Longrightarrow:\boxed{\sf{y=c-a}}[/tex]

Therefore, the correct answer is y=c-a.

I hope this helps you! Let me know if my answer is wrong or not.

The linear function f(x)=ax+b is graphed in the coordinate plane. Two equal intervals are compared,[-3,3] and [9,15]. What conclusion can be drawn from each interval of 6?

Answers

The linear function f(x) = ax + b has a constant rate

The conclusion from the each interval is that the function values changes at a constant rate

How to determine the conclusion?

The linear function is given as:

f(x) = ax + b

The interval is given as:

[-3,3] and [9,15]

Since the function is a linear function, the conclusion from the each interval is that the function values changes at a constant rate

Read more about linear functions at:

https://brainly.com/question/14323743

A betta fish tank is in the shape of a cube. The volume of the fish tank is 216
cubic inches. What is the edge length, in inches, of the fish tank?
6)
Type the number in the box.
inches

Answers

To find this answer, we need to understand how to get the Volume.

We get the volume by the formula: L x W x H

L = Length

W = Width

H = Height

So, we can see that there are 3 units required to know for getting the volume, if that's true, then, to do the inverse operation of finding the volume. That can be done by using thie formula:

[tex]V=a^3[/tex]

a= edge

Solving for a

a=V⅓=216⅓=6

So hence, each of the Cube shaped Fish Tank is 6in³

now, let's look at how to find whether our answer is right or wrong

6x6=36

36x6=216

So hence, our answer is right!

-------------------------------------------------------------------------------------------------------"The beautiful thing about learning is that no one can take it away from you!" B.B.King

Thanks!

Mark me brainliest!

Have a GREAT AWESOME day!

~ms115~

:D

The length of a betta fish tank that has a shape of a cube and volume of 216 cubic inches is 6 inches.

Volume of a cube

A cube has all its sides equal to each other. Therefore,

volume of cube = L³

where

L = length

Therefore,

L = >

volume = 216 cubic inches

volume of cube = L³

216 = L³

cube root both sides

∛216 = ∛L³

L = ∛216

L = 6 inches

learn more on volume here: https://brainly.com/question/10961243

What is the perimeter of the polygon below?

Answers

The answer to that is 20.5

The perimeter of the polygon will be 30 units. Then the correct option is D.

What is the perimeter of the regular polygon?

All the sides of the regular polygon are congruent to each other. The perimeter of the regular polygon of n sides will be the product of the number of the side and the side length of the regular polygon.

P = (Side length) x n

The sides length of the polygon is 10 units, 5 units, 3 units, 2.5 units, 7 units, and 2.5 units.

Then the perimeter of the polygon is given as,

P = 10 + 5 + 3 + 2.5 + 7 + 2.5

P = 30 units

The perimeter of the polygon will be 30 units. Then the correct option is D.

More about the perimeter of the regular polygon link is given below.

https://brainly.com/question/10885363

#SPJ7

if the price of small box is $20 and the price of large box is $45. what is the greatest amount of money he received when he sells all the boxes he has made? For this amount of money how many boxes of each size did he make​

Answers

Answer:

in order to know the money that he receive first the type and amount of box must be described unless the question may not have answer.

Esmeralda has a magnet in the shape of a square. Each side of the magnet is 3 inches long. What is the perimeter of her magnet?​

Answers

Answer:

12 inches

Step-by-step explanation:

This is a square of sidee length 3 inches.  As there are four (equal) sides, the perimeter of this square magnet is 4(3 inches), or 12 inches.

4. A plumber charges $65 per hour to fix a leak. What is an equation that relates the total cost of a leak repair c to the number of hours, h, it takes the
plumber to fix the leak? Show work

Answers

Answer:

c = 65h.

Step-by-step explanation:

cost = rate per hour * hours worked:

c = 65 * h .


Which correctly shows this equation
solved for w?

Answers

A because the W needs to be alone because that is what you are trying to find. If you have to solve for Y you would have the y alone and same if you had x. x would be alone on the left

The equation solved for w is:

w = A/l

The correct option is A.

What is an equation?

A pair of algebraic equations with the equal symbol (=) in the center and the same value are referred to as an equation.

Starting with the equation:

A/l = lw/l

We can simplify it by multiplying both sides by l:

A = lw

To solve for w, we need to isolate it on one side of the equation.

We can do this by dividing both sides by l:

A/l = w

Therefore, the required equation is A/l = w.

To learn more about the equation;

https://brainly.com/question/12788590

#SPJ7

1/2 of 4/5 as a fraction

Answers

Answer:

4/10 is the answer

( 1 * 4 ) / ( 4 * 5 ) = 4 / 10

4 / 10 = 2 / 5

what is f(x)25x2+9x+10

Answers

The answer should be f(x)=9x + 60

it’s urgent please help

Answers

Answer:

  x = 5.4

Step-by-step explanation:

The marked sides are the hypotenuse and the side adjacent to the marked angle. That suggests the appropriate trig function is the cosine:

  Cos = Adjacent/Hypotenuse

  cos(19°) = 5.1/x

  x = 5.1/cos(19°) . . . . . multiply by x/cos(19°)

  x ≈ 5.4

find the measures of
Find the measure of
Find the measure of


Find the measure of

Answers

Answer:

15.

If we look at angle BOD, that would form a right angle (90°), given that one part of that whole angle is 30°, the other would be;

30° + x = 90°

-30        -30

x = 60°, therefore ∠BOC = 60°.

If we take a look at angle EOC, we can see that that angle is a straight angle (180°), we can also see that a right angle (90°) is a part of that angle alongside the angle we just previously found (60°). So all of those angles plus the unknown angle (AOE) which we will consider 'x' summed up would result in 180 degrees.

Now we set up the equation;

90° + 60° + x = 180°

150 + x = 180

-150       -150

x = 30°, therefore ∠AOE = 30°.

16.

The sum of the interior angles in a triangle will always equal 180°. (We can also confirm this using the formula (n - 2) x 180.)

Given two of the angles, we must add them and the unknown angle(D) which we will consider 'x' to make it result in 180°.

Now we set up the equation;

55° + 18° + x = 180°

73 + x = 180

-73         -73

x = 107°, therefore ∠EDF = 107°.

17.

To find angle P, we must first find the supplement of 34° because 34° and the angle beside forms a straight angle (180°).

Set up an equation;

34 + x = 180

-34         -34

x = 146°, now that we've found the supplement, we add this supplement with the other given angle (23°) because all three angles (unknown angle which we will consider x + 23 + 146) will equal 180°(sum of interior angles of triangle).

146° + 23° + x = 180°

169 + x = 180

-169        -169

x = 11°, therefore ∠QPR = 11°.

18.

Seeing that the bigger triangle has a 90° angle (indicated with a square), and two other equal angles(indicated with the two lines on both legs of the big triangle), we solve for those two missing equal angles in the bigger triangle which we will then use to solve for the smaller triangle's angle.

2x + 90° = 180°

-90            -90

2x = 90

/2     /2

x = 45°, so now we know the two angles in the bigger triangle excluding the right angle.

One of those equal angles is vertical to the smaller triangle, and vertical angles are congruent.

Hence, the angle vertical to the bigger triangle in the smaller triangle will be 45°.

Now we solve for ∠CDE.

Add the two angles and the missing angle to equal 180°.

Set up the equation;

86° + 45° + x = 180°

131 + x = 180

-131        -131

x = 49°, so ∠CDE = 49°.

What method would you choose to solve the equation 2x 2squared – 7 = 9? Explain why you chose this method.

Answers

Answer:

Step-by-step explanation:

(2x2squared-7)2squared=9 2squared

4x4squared-28x2squared+49=81

4x4squared-28x2squared=81-49

4x4squared-28x2squared=32

Other Questions
Why couldn't Mapes get up time after time after being shot? why is it important to know the evolution of media Has the United States always been a country with many formal organizations Whoever gets it right can have brainliest, but answer ASAP! the dimensions of a cuboidal block are 2.5 m * 2m * 1.2m its weight 900 n what is the minimum presseure exerted by the block on the surface of its support.? A company that deals in breakfast cereals advertises that its products contain all the essential nutrients necessary for the proper growth and development of children. This is an example of: Write a short story about HIV and AIDS. One side of Dylan'ssquare bedroom is 10feet longWhat is thetotal perimeter ofDylan's bedroom? Question 2 Before Boggs dies, he gives Katniss one final order: "Don't trust them. Don't go back. Kill Peeta. Do what you came to do." Interpret Boggs's final words. What does he mean? Who shouldn't Katniss trust? The rebels? Coin? Her squad? Why? Your response should be four complete sentences or longer. What were the author's purposes in writing It's Our World, Too!: Young People Who Are Making a Difference? Checkall that applyVAto entertain readers with the antics of young high school students1 to persuade readers that all students are equal, regardless of raceto inform readers about the ways young people are changing the worldUD to entertain readers with a story about high schoolers who made a changetO persuade readers that racism still exists in today's societyto inform readers about different ways that changes can be made What action prompted President Roosevelt to suggest the plan referred to in the cartoon (1) The Supreme Court had declared prominent New Deal programs unconstitutional (2) Roosevelt had lost the popular vote in the last presidential election. ( 3) Congress had passed a bill reducing the size of the Supreme Court (4) Roosevelt had announced an end to the New Deal. Round your answer to the nearest hundredth. Alan bought 120 pens at the cost of 2 per pen. He sold 72 of them for 2.50 per pen and the remaining at the price of 2 per pen. Find his percentage profit or loss. In a discount basis loan, interest is "pald" at the time the loan is made by deducting the Interest amount from the loanamount.TrueFalse dalandan and mangoes are both fruits, which means they have seeds inside of them. each has a skin, dalandan skins are thichker and easier to peel than that of mageos, but both are delicious if a=6, evaluate the following expression:10+a/4 What is the answer elmo wants it help its for calculus Examine the following piecewise function.Which statements are true?Select all that apply.1.The function is increasing over the interval 2x3.2.The function is increasing over the interval 2x2.3.The function is increasing over the interval 8x2.4.The function is decreasing over the interval 3x7.5.The function is constant over the interval 2x2. What is the result when the number 51 is decreased by 75%?